Can the sum of the first $p$ factorials ever be a perfect power for $ p>3 $?












1












$begingroup$



Has $$sum_{j=1}^p j!=q^r$$ , where q,p,r are positive integers, and r > 1 , a solution ?




I solved partially, if r is even, then RHS is a perfect square, and there is no doubt in that. Therefore, the left side must be a perfect square as well.
But for p>3, the last digit of LHS is 3 which is not the last digit of any square. Hence, p<4, and hence manually checking, only solutions are p =1,3.
But i cannot generalize when r is odd.
Any solution to the next part would be helpful!










share|cite|improve this question











$endgroup$

















    1












    $begingroup$



    Has $$sum_{j=1}^p j!=q^r$$ , where q,p,r are positive integers, and r > 1 , a solution ?




    I solved partially, if r is even, then RHS is a perfect square, and there is no doubt in that. Therefore, the left side must be a perfect square as well.
    But for p>3, the last digit of LHS is 3 which is not the last digit of any square. Hence, p<4, and hence manually checking, only solutions are p =1,3.
    But i cannot generalize when r is odd.
    Any solution to the next part would be helpful!










    share|cite|improve this question











    $endgroup$















      1












      1








      1





      $begingroup$



      Has $$sum_{j=1}^p j!=q^r$$ , where q,p,r are positive integers, and r > 1 , a solution ?




      I solved partially, if r is even, then RHS is a perfect square, and there is no doubt in that. Therefore, the left side must be a perfect square as well.
      But for p>3, the last digit of LHS is 3 which is not the last digit of any square. Hence, p<4, and hence manually checking, only solutions are p =1,3.
      But i cannot generalize when r is odd.
      Any solution to the next part would be helpful!










      share|cite|improve this question











      $endgroup$





      Has $$sum_{j=1}^p j!=q^r$$ , where q,p,r are positive integers, and r > 1 , a solution ?




      I solved partially, if r is even, then RHS is a perfect square, and there is no doubt in that. Therefore, the left side must be a perfect square as well.
      But for p>3, the last digit of LHS is 3 which is not the last digit of any square. Hence, p<4, and hence manually checking, only solutions are p =1,3.
      But i cannot generalize when r is odd.
      Any solution to the next part would be helpful!







      linear-algebra factorial






      share|cite|improve this question















      share|cite|improve this question













      share|cite|improve this question




      share|cite|improve this question








      edited Jan 19 at 14:29









      Peter

      47.6k1039131




      47.6k1039131










      asked Jan 19 at 14:13







      user636268





























          1 Answer
          1






          active

          oldest

          votes


















          1












          $begingroup$

          Let $S(p)=sum_{j=1}^pj!$ denote your sum.



          Claim: For $p≥8$ $v_3(S(p))=2$



          (Here, as usual, for a prime $q$ and natural number $n$, $v_q(n)$ denotes the order to which $q$ divides $n$. Thus, $v_3(18)=2$ for example.).



          Pf: One computes that $S(8)=3^2times 11times 467$ and from there after you have at least three factors of $3$ in each summand.



          It follows immediately that, at least for $p≥8$ your sum could not be any power of degree greater than $2$. As you have already addressed the case of squares, we are done (after a simple search for small $p$).






          share|cite|improve this answer









          $endgroup$









          • 1




            $begingroup$
            You answered despite of the demanding style ?
            $endgroup$
            – Peter
            Jan 19 at 14:43












          • $begingroup$
            @Peter I didn't look at the comments, you are right about the tone.
            $endgroup$
            – lulu
            Jan 19 at 14:44






          • 1




            $begingroup$
            @lulu I think users should not be encouraged to get answers this way. So, my suggestion is to delete the answer , although it is a very nice answer.
            $endgroup$
            – Peter
            Jan 19 at 14:50






          • 2




            $begingroup$
            @Peter I'm on the fence. The question itself is very well posed, I thought. The OP addresses a major subcase of the problem. The tone of the comments is bad, and had I read them I would have stopped thinking about the problem. As it is, I think the question is mathematically interesting so I'm going to leave the solution up, though I do think you have a point.
            $endgroup$
            – lulu
            Jan 19 at 15:22










          • $begingroup$
            @lulu I think you are right, and the elegant solution is surely useful also for other users.
            $endgroup$
            – Peter
            Jan 19 at 15:23











          Your Answer





          StackExchange.ifUsing("editor", function () {
          return StackExchange.using("mathjaxEditing", function () {
          StackExchange.MarkdownEditor.creationCallbacks.add(function (editor, postfix) {
          StackExchange.mathjaxEditing.prepareWmdForMathJax(editor, postfix, [["$", "$"], ["\\(","\\)"]]);
          });
          });
          }, "mathjax-editing");

          StackExchange.ready(function() {
          var channelOptions = {
          tags: "".split(" "),
          id: "69"
          };
          initTagRenderer("".split(" "), "".split(" "), channelOptions);

          StackExchange.using("externalEditor", function() {
          // Have to fire editor after snippets, if snippets enabled
          if (StackExchange.settings.snippets.snippetsEnabled) {
          StackExchange.using("snippets", function() {
          createEditor();
          });
          }
          else {
          createEditor();
          }
          });

          function createEditor() {
          StackExchange.prepareEditor({
          heartbeatType: 'answer',
          autoActivateHeartbeat: false,
          convertImagesToLinks: true,
          noModals: true,
          showLowRepImageUploadWarning: true,
          reputationToPostImages: 10,
          bindNavPrevention: true,
          postfix: "",
          imageUploader: {
          brandingHtml: "Powered by u003ca class="icon-imgur-white" href="https://imgur.com/"u003eu003c/au003e",
          contentPolicyHtml: "User contributions licensed under u003ca href="https://creativecommons.org/licenses/by-sa/3.0/"u003ecc by-sa 3.0 with attribution requiredu003c/au003e u003ca href="https://stackoverflow.com/legal/content-policy"u003e(content policy)u003c/au003e",
          allowUrls: true
          },
          noCode: true, onDemand: true,
          discardSelector: ".discard-answer"
          ,immediatelyShowMarkdownHelp:true
          });


          }
          });














          draft saved

          draft discarded


















          StackExchange.ready(
          function () {
          StackExchange.openid.initPostLogin('.new-post-login', 'https%3a%2f%2fmath.stackexchange.com%2fquestions%2f3079389%2fcan-the-sum-of-the-first-p-factorials-ever-be-a-perfect-power-for-p3%23new-answer', 'question_page');
          }
          );

          Post as a guest















          Required, but never shown
























          1 Answer
          1






          active

          oldest

          votes








          1 Answer
          1






          active

          oldest

          votes









          active

          oldest

          votes






          active

          oldest

          votes









          1












          $begingroup$

          Let $S(p)=sum_{j=1}^pj!$ denote your sum.



          Claim: For $p≥8$ $v_3(S(p))=2$



          (Here, as usual, for a prime $q$ and natural number $n$, $v_q(n)$ denotes the order to which $q$ divides $n$. Thus, $v_3(18)=2$ for example.).



          Pf: One computes that $S(8)=3^2times 11times 467$ and from there after you have at least three factors of $3$ in each summand.



          It follows immediately that, at least for $p≥8$ your sum could not be any power of degree greater than $2$. As you have already addressed the case of squares, we are done (after a simple search for small $p$).






          share|cite|improve this answer









          $endgroup$









          • 1




            $begingroup$
            You answered despite of the demanding style ?
            $endgroup$
            – Peter
            Jan 19 at 14:43












          • $begingroup$
            @Peter I didn't look at the comments, you are right about the tone.
            $endgroup$
            – lulu
            Jan 19 at 14:44






          • 1




            $begingroup$
            @lulu I think users should not be encouraged to get answers this way. So, my suggestion is to delete the answer , although it is a very nice answer.
            $endgroup$
            – Peter
            Jan 19 at 14:50






          • 2




            $begingroup$
            @Peter I'm on the fence. The question itself is very well posed, I thought. The OP addresses a major subcase of the problem. The tone of the comments is bad, and had I read them I would have stopped thinking about the problem. As it is, I think the question is mathematically interesting so I'm going to leave the solution up, though I do think you have a point.
            $endgroup$
            – lulu
            Jan 19 at 15:22










          • $begingroup$
            @lulu I think you are right, and the elegant solution is surely useful also for other users.
            $endgroup$
            – Peter
            Jan 19 at 15:23
















          1












          $begingroup$

          Let $S(p)=sum_{j=1}^pj!$ denote your sum.



          Claim: For $p≥8$ $v_3(S(p))=2$



          (Here, as usual, for a prime $q$ and natural number $n$, $v_q(n)$ denotes the order to which $q$ divides $n$. Thus, $v_3(18)=2$ for example.).



          Pf: One computes that $S(8)=3^2times 11times 467$ and from there after you have at least three factors of $3$ in each summand.



          It follows immediately that, at least for $p≥8$ your sum could not be any power of degree greater than $2$. As you have already addressed the case of squares, we are done (after a simple search for small $p$).






          share|cite|improve this answer









          $endgroup$









          • 1




            $begingroup$
            You answered despite of the demanding style ?
            $endgroup$
            – Peter
            Jan 19 at 14:43












          • $begingroup$
            @Peter I didn't look at the comments, you are right about the tone.
            $endgroup$
            – lulu
            Jan 19 at 14:44






          • 1




            $begingroup$
            @lulu I think users should not be encouraged to get answers this way. So, my suggestion is to delete the answer , although it is a very nice answer.
            $endgroup$
            – Peter
            Jan 19 at 14:50






          • 2




            $begingroup$
            @Peter I'm on the fence. The question itself is very well posed, I thought. The OP addresses a major subcase of the problem. The tone of the comments is bad, and had I read them I would have stopped thinking about the problem. As it is, I think the question is mathematically interesting so I'm going to leave the solution up, though I do think you have a point.
            $endgroup$
            – lulu
            Jan 19 at 15:22










          • $begingroup$
            @lulu I think you are right, and the elegant solution is surely useful also for other users.
            $endgroup$
            – Peter
            Jan 19 at 15:23














          1












          1








          1





          $begingroup$

          Let $S(p)=sum_{j=1}^pj!$ denote your sum.



          Claim: For $p≥8$ $v_3(S(p))=2$



          (Here, as usual, for a prime $q$ and natural number $n$, $v_q(n)$ denotes the order to which $q$ divides $n$. Thus, $v_3(18)=2$ for example.).



          Pf: One computes that $S(8)=3^2times 11times 467$ and from there after you have at least three factors of $3$ in each summand.



          It follows immediately that, at least for $p≥8$ your sum could not be any power of degree greater than $2$. As you have already addressed the case of squares, we are done (after a simple search for small $p$).






          share|cite|improve this answer









          $endgroup$



          Let $S(p)=sum_{j=1}^pj!$ denote your sum.



          Claim: For $p≥8$ $v_3(S(p))=2$



          (Here, as usual, for a prime $q$ and natural number $n$, $v_q(n)$ denotes the order to which $q$ divides $n$. Thus, $v_3(18)=2$ for example.).



          Pf: One computes that $S(8)=3^2times 11times 467$ and from there after you have at least three factors of $3$ in each summand.



          It follows immediately that, at least for $p≥8$ your sum could not be any power of degree greater than $2$. As you have already addressed the case of squares, we are done (after a simple search for small $p$).







          share|cite|improve this answer












          share|cite|improve this answer



          share|cite|improve this answer










          answered Jan 19 at 14:40









          lulululu

          41.6k24979




          41.6k24979








          • 1




            $begingroup$
            You answered despite of the demanding style ?
            $endgroup$
            – Peter
            Jan 19 at 14:43












          • $begingroup$
            @Peter I didn't look at the comments, you are right about the tone.
            $endgroup$
            – lulu
            Jan 19 at 14:44






          • 1




            $begingroup$
            @lulu I think users should not be encouraged to get answers this way. So, my suggestion is to delete the answer , although it is a very nice answer.
            $endgroup$
            – Peter
            Jan 19 at 14:50






          • 2




            $begingroup$
            @Peter I'm on the fence. The question itself is very well posed, I thought. The OP addresses a major subcase of the problem. The tone of the comments is bad, and had I read them I would have stopped thinking about the problem. As it is, I think the question is mathematically interesting so I'm going to leave the solution up, though I do think you have a point.
            $endgroup$
            – lulu
            Jan 19 at 15:22










          • $begingroup$
            @lulu I think you are right, and the elegant solution is surely useful also for other users.
            $endgroup$
            – Peter
            Jan 19 at 15:23














          • 1




            $begingroup$
            You answered despite of the demanding style ?
            $endgroup$
            – Peter
            Jan 19 at 14:43












          • $begingroup$
            @Peter I didn't look at the comments, you are right about the tone.
            $endgroup$
            – lulu
            Jan 19 at 14:44






          • 1




            $begingroup$
            @lulu I think users should not be encouraged to get answers this way. So, my suggestion is to delete the answer , although it is a very nice answer.
            $endgroup$
            – Peter
            Jan 19 at 14:50






          • 2




            $begingroup$
            @Peter I'm on the fence. The question itself is very well posed, I thought. The OP addresses a major subcase of the problem. The tone of the comments is bad, and had I read them I would have stopped thinking about the problem. As it is, I think the question is mathematically interesting so I'm going to leave the solution up, though I do think you have a point.
            $endgroup$
            – lulu
            Jan 19 at 15:22










          • $begingroup$
            @lulu I think you are right, and the elegant solution is surely useful also for other users.
            $endgroup$
            – Peter
            Jan 19 at 15:23








          1




          1




          $begingroup$
          You answered despite of the demanding style ?
          $endgroup$
          – Peter
          Jan 19 at 14:43






          $begingroup$
          You answered despite of the demanding style ?
          $endgroup$
          – Peter
          Jan 19 at 14:43














          $begingroup$
          @Peter I didn't look at the comments, you are right about the tone.
          $endgroup$
          – lulu
          Jan 19 at 14:44




          $begingroup$
          @Peter I didn't look at the comments, you are right about the tone.
          $endgroup$
          – lulu
          Jan 19 at 14:44




          1




          1




          $begingroup$
          @lulu I think users should not be encouraged to get answers this way. So, my suggestion is to delete the answer , although it is a very nice answer.
          $endgroup$
          – Peter
          Jan 19 at 14:50




          $begingroup$
          @lulu I think users should not be encouraged to get answers this way. So, my suggestion is to delete the answer , although it is a very nice answer.
          $endgroup$
          – Peter
          Jan 19 at 14:50




          2




          2




          $begingroup$
          @Peter I'm on the fence. The question itself is very well posed, I thought. The OP addresses a major subcase of the problem. The tone of the comments is bad, and had I read them I would have stopped thinking about the problem. As it is, I think the question is mathematically interesting so I'm going to leave the solution up, though I do think you have a point.
          $endgroup$
          – lulu
          Jan 19 at 15:22




          $begingroup$
          @Peter I'm on the fence. The question itself is very well posed, I thought. The OP addresses a major subcase of the problem. The tone of the comments is bad, and had I read them I would have stopped thinking about the problem. As it is, I think the question is mathematically interesting so I'm going to leave the solution up, though I do think you have a point.
          $endgroup$
          – lulu
          Jan 19 at 15:22












          $begingroup$
          @lulu I think you are right, and the elegant solution is surely useful also for other users.
          $endgroup$
          – Peter
          Jan 19 at 15:23




          $begingroup$
          @lulu I think you are right, and the elegant solution is surely useful also for other users.
          $endgroup$
          – Peter
          Jan 19 at 15:23


















          draft saved

          draft discarded




















































          Thanks for contributing an answer to Mathematics Stack Exchange!


          • Please be sure to answer the question. Provide details and share your research!

          But avoid



          • Asking for help, clarification, or responding to other answers.

          • Making statements based on opinion; back them up with references or personal experience.


          Use MathJax to format equations. MathJax reference.


          To learn more, see our tips on writing great answers.




          draft saved


          draft discarded














          StackExchange.ready(
          function () {
          StackExchange.openid.initPostLogin('.new-post-login', 'https%3a%2f%2fmath.stackexchange.com%2fquestions%2f3079389%2fcan-the-sum-of-the-first-p-factorials-ever-be-a-perfect-power-for-p3%23new-answer', 'question_page');
          }
          );

          Post as a guest















          Required, but never shown





















































          Required, but never shown














          Required, but never shown












          Required, but never shown







          Required, but never shown

































          Required, but never shown














          Required, but never shown












          Required, but never shown







          Required, but never shown







          Popular posts from this blog

          Mario Kart Wii

          What does “Dominus providebit” mean?

          Antonio Litta Visconti Arese